Determine the generator of an ideal of ring of integers Announcing the arrival of Valued Associate #679: Cesar Manara Planned maintenance scheduled April 23, 2019 at 23:30 UTC (7:30pm US/Eastern)Non-zero prime ideals in the ring of all algebraic integersConstructing Idempotent Generator of Idempotent Idealideal and ideal classes in the ring of integers.Is the minimal number of generators of an ideal the rank of the ideal as a free $mathbb Z$-module?Number of generators of an ideal of the polynomial ring over a fieldIs the ratio of the norms of generators in an ideal well defined?Finding ideal generator in real quadratic fieldsIdeal Class Group Calculation: How to conclude the classes of two ideals are distinctIs there a non-constant $h in mathbbC[x_1 , dots , x_n ]$ that divides every element of this given ideal?Show that an ideal of the ring of integers of a real number field is not principal

Where is Bhagavad Gita referred to as Hari Gita?

Does using the Inspiration rules for character defects encourage My Guy Syndrome?

What could prevent concentrated local exploration?

How do I deal with an erroneously large refund?

Why does BitLocker not use RSA?

What were wait-states, and why was it only an issue for PCs?

Meaning of "Not holding on that level of emuna/bitachon"

What is the ongoing value of the Kanban board to the developers as opposed to management

How can I introduce the names of fantasy creatures to the reader?

When speaking, how do you change your mind mid-sentence?

Who can become a wight?

Putting Ant-Man on house arrest

Can the van der Waals coefficients be negative in the van der Waals equation for real gases?

A German immigrant ancestor has a "Registration Affidavit of Alien Enemy" on file. What does that mean exactly?

Raising a bilingual kid. When should we introduce the majority language?

Reflections in a Square

What is the definining line between a helicopter and a drone a person can ride in?

Can gravitational waves pass through a black hole?

Does Prince Arnaud cause someone holding the Princess to lose?

Trying to enter the Fox's den

What helicopter has the most rotor blades?

Does GDPR cover the collection of data by websites that crawl the web and resell user data

Why are two-digit numbers in Jonathan Swift's "Gulliver's Travels" (1726) written in "German style"?

How to get a single big right brace?



Determine the generator of an ideal of ring of integers



Announcing the arrival of Valued Associate #679: Cesar Manara
Planned maintenance scheduled April 23, 2019 at 23:30 UTC (7:30pm US/Eastern)Non-zero prime ideals in the ring of all algebraic integersConstructing Idempotent Generator of Idempotent Idealideal and ideal classes in the ring of integers.Is the minimal number of generators of an ideal the rank of the ideal as a free $mathbb Z$-module?Number of generators of an ideal of the polynomial ring over a fieldIs the ratio of the norms of generators in an ideal well defined?Finding ideal generator in real quadratic fieldsIdeal Class Group Calculation: How to conclude the classes of two ideals are distinctIs there a non-constant $h in mathbbC[x_1 , dots , x_n ]$ that divides every element of this given ideal?Show that an ideal of the ring of integers of a real number field is not principal










4












$begingroup$


I am trying to find the generators of the ideal $(3)$ in the ring of integers of $mathbbQ[sqrt-83]$ the ring of integers is $mathbbZleft[frac1+sqrt-832right]$ I evaluated the Minkowski constant and it is $2/pi sqrt83 sim 5.8;$ then I checked the minimal polynomial of $x^2-x+84/2, $ which is reducible module $3,$ hence the ideal $(3)$ is generated by two elements, right? Did I miss something? I want to say the ring of integers is not a UFD.










share|cite|improve this question











$endgroup$
















    4












    $begingroup$


    I am trying to find the generators of the ideal $(3)$ in the ring of integers of $mathbbQ[sqrt-83]$ the ring of integers is $mathbbZleft[frac1+sqrt-832right]$ I evaluated the Minkowski constant and it is $2/pi sqrt83 sim 5.8;$ then I checked the minimal polynomial of $x^2-x+84/2, $ which is reducible module $3,$ hence the ideal $(3)$ is generated by two elements, right? Did I miss something? I want to say the ring of integers is not a UFD.










    share|cite|improve this question











    $endgroup$














      4












      4








      4





      $begingroup$


      I am trying to find the generators of the ideal $(3)$ in the ring of integers of $mathbbQ[sqrt-83]$ the ring of integers is $mathbbZleft[frac1+sqrt-832right]$ I evaluated the Minkowski constant and it is $2/pi sqrt83 sim 5.8;$ then I checked the minimal polynomial of $x^2-x+84/2, $ which is reducible module $3,$ hence the ideal $(3)$ is generated by two elements, right? Did I miss something? I want to say the ring of integers is not a UFD.










      share|cite|improve this question











      $endgroup$




      I am trying to find the generators of the ideal $(3)$ in the ring of integers of $mathbbQ[sqrt-83]$ the ring of integers is $mathbbZleft[frac1+sqrt-832right]$ I evaluated the Minkowski constant and it is $2/pi sqrt83 sim 5.8;$ then I checked the minimal polynomial of $x^2-x+84/2, $ which is reducible module $3,$ hence the ideal $(3)$ is generated by two elements, right? Did I miss something? I want to say the ring of integers is not a UFD.







      ring-theory algebraic-number-theory ideal-class-group






      share|cite|improve this question















      share|cite|improve this question













      share|cite|improve this question




      share|cite|improve this question








      edited 5 hours ago









      J. W. Tanner

      5,1351520




      5,1351520










      asked 6 hours ago









      AmeryrAmeryr

      768311




      768311




















          2 Answers
          2






          active

          oldest

          votes


















          4












          $begingroup$

          You seem to have touched upon several different ideas here.



          Generators of the ideal $(3)$. Usually, when you talk about the generators of $(3)$, you mean its generators as an abelian group.



          Define $theta := tfrac 1 2 (1 + sqrt-83)$. We know that the ring of integers $mathbb Z[theta]$ is generated by $1, theta $ as an abelian group, so $(3)$ is generated by $3, 3theta $ as an abelian group.



          But having read the remainder of your question, it looks like this is not what you're after! What you're really after are the generators of the ideal class group for $mathbb Z[theta]$...



          Minkowski constant. The fact that the Minkowski constant is $5.8$ implies that the ideal class group is generated by prime ideals that are factors of $(2)$ or $(3)$.



          Dedekind's criterion. Dedekind's criterion is a way of factorising $(3)$ as a product of primes.



          As you pointed out, we have the factorisation
          $$ x^2 - x + frac 842 equiv x(x-1) mod 3,$$



          so Dedekind's criterion says that
          $$ (3) = (3, theta)(3, theta - 1)$$
          is the prime factorisation of $(3)$ in $mathbb Z[theta]$.



          Whether $mathbb Z[theta]$ is a UFD. This is equivalent to asking whether the ideal class group is trivial, i.e. whether all ideals are principal.



          Why don't we check whether $(3, theta)$ is principal? The answer must be "no". Note that $(3, theta)$ has norm $3$. If it was principal, then there would exist $x, y in mathbb Z$ such that $(3, theta) = (x + ytheta)$. But then
          $$ 3 = N(3, theta) = N(x + ytheta) = (x + tfrac 1 2 y)^2 + 83(tfrac 1 2 y)^2,$$
          which is impossible.



          So $mathbb Z[theta]$ is not a principal ideal domain, and hence, is not a unique factorisation domain.






          share|cite|improve this answer









          $endgroup$












          • $begingroup$
            Thanks that helpful, that what I was looking for
            $endgroup$
            – Ameryr
            4 hours ago



















          3












          $begingroup$

          The minimal polynomial of $alpha=frac1+sqrt-832$ is $f = x^2 - x + 84/4 = x^2 - x + 21$. Modulo $3$ this factors as $$f equiv x^2 - x = x(x-1) pmod 3,$$
          so by the Kummer-Dedekind theorem the ideal $(3)$ factors into primes in $mathbbZbigg[frac1+sqrt-832bigg]$ as $(3) = (3, alpha)(3, alpha-1)$.



          The ideal $(3)$ is principal, generated by $3$. You can show that the prime factors are not principal, e.g. using the field norm:



          If $(3,alpha) = (beta)$ then $N(beta)$ divides both $N(3) = 3^2$ and $N(alpha) = f(0) = 21 = 3cdot 7$, so $N(beta) = 3$.



          If we write $beta = a+balpha$ then $$N(beta)=(a+balpha)(a+b(1-alpha)) = ldots = a^2+ab + 21b^2.$$



          The equation $a^2+ab + 21b^2 = 3$ is an ellipse in the $(a,b)$-plane without integral points, so there is no such $beta$.






          share|cite|improve this answer









          $endgroup$








          • 1




            $begingroup$
            How could you determine that the ellipse has no integral points?
            $endgroup$
            – Ameryr
            4 hours ago






          • 1




            $begingroup$
            I plotted it (with a grid in the background). It's a very small ellipse, so there are very few candidates for integral points, and you can see that it misses them all.
            $endgroup$
            – Ricardo Buring
            4 hours ago











          Your Answer








          StackExchange.ready(function()
          var channelOptions =
          tags: "".split(" "),
          id: "69"
          ;
          initTagRenderer("".split(" "), "".split(" "), channelOptions);

          StackExchange.using("externalEditor", function()
          // Have to fire editor after snippets, if snippets enabled
          if (StackExchange.settings.snippets.snippetsEnabled)
          StackExchange.using("snippets", function()
          createEditor();
          );

          else
          createEditor();

          );

          function createEditor()
          StackExchange.prepareEditor(
          heartbeatType: 'answer',
          autoActivateHeartbeat: false,
          convertImagesToLinks: true,
          noModals: true,
          showLowRepImageUploadWarning: true,
          reputationToPostImages: 10,
          bindNavPrevention: true,
          postfix: "",
          imageUploader:
          brandingHtml: "Powered by u003ca class="icon-imgur-white" href="https://imgur.com/"u003eu003c/au003e",
          contentPolicyHtml: "User contributions licensed under u003ca href="https://creativecommons.org/licenses/by-sa/3.0/"u003ecc by-sa 3.0 with attribution requiredu003c/au003e u003ca href="https://stackoverflow.com/legal/content-policy"u003e(content policy)u003c/au003e",
          allowUrls: true
          ,
          noCode: true, onDemand: true,
          discardSelector: ".discard-answer"
          ,immediatelyShowMarkdownHelp:true
          );



          );













          draft saved

          draft discarded


















          StackExchange.ready(
          function ()
          StackExchange.openid.initPostLogin('.new-post-login', 'https%3a%2f%2fmath.stackexchange.com%2fquestions%2f3197383%2fdetermine-the-generator-of-an-ideal-of-ring-of-integers%23new-answer', 'question_page');

          );

          Post as a guest















          Required, but never shown

























          2 Answers
          2






          active

          oldest

          votes








          2 Answers
          2






          active

          oldest

          votes









          active

          oldest

          votes






          active

          oldest

          votes









          4












          $begingroup$

          You seem to have touched upon several different ideas here.



          Generators of the ideal $(3)$. Usually, when you talk about the generators of $(3)$, you mean its generators as an abelian group.



          Define $theta := tfrac 1 2 (1 + sqrt-83)$. We know that the ring of integers $mathbb Z[theta]$ is generated by $1, theta $ as an abelian group, so $(3)$ is generated by $3, 3theta $ as an abelian group.



          But having read the remainder of your question, it looks like this is not what you're after! What you're really after are the generators of the ideal class group for $mathbb Z[theta]$...



          Minkowski constant. The fact that the Minkowski constant is $5.8$ implies that the ideal class group is generated by prime ideals that are factors of $(2)$ or $(3)$.



          Dedekind's criterion. Dedekind's criterion is a way of factorising $(3)$ as a product of primes.



          As you pointed out, we have the factorisation
          $$ x^2 - x + frac 842 equiv x(x-1) mod 3,$$



          so Dedekind's criterion says that
          $$ (3) = (3, theta)(3, theta - 1)$$
          is the prime factorisation of $(3)$ in $mathbb Z[theta]$.



          Whether $mathbb Z[theta]$ is a UFD. This is equivalent to asking whether the ideal class group is trivial, i.e. whether all ideals are principal.



          Why don't we check whether $(3, theta)$ is principal? The answer must be "no". Note that $(3, theta)$ has norm $3$. If it was principal, then there would exist $x, y in mathbb Z$ such that $(3, theta) = (x + ytheta)$. But then
          $$ 3 = N(3, theta) = N(x + ytheta) = (x + tfrac 1 2 y)^2 + 83(tfrac 1 2 y)^2,$$
          which is impossible.



          So $mathbb Z[theta]$ is not a principal ideal domain, and hence, is not a unique factorisation domain.






          share|cite|improve this answer









          $endgroup$












          • $begingroup$
            Thanks that helpful, that what I was looking for
            $endgroup$
            – Ameryr
            4 hours ago
















          4












          $begingroup$

          You seem to have touched upon several different ideas here.



          Generators of the ideal $(3)$. Usually, when you talk about the generators of $(3)$, you mean its generators as an abelian group.



          Define $theta := tfrac 1 2 (1 + sqrt-83)$. We know that the ring of integers $mathbb Z[theta]$ is generated by $1, theta $ as an abelian group, so $(3)$ is generated by $3, 3theta $ as an abelian group.



          But having read the remainder of your question, it looks like this is not what you're after! What you're really after are the generators of the ideal class group for $mathbb Z[theta]$...



          Minkowski constant. The fact that the Minkowski constant is $5.8$ implies that the ideal class group is generated by prime ideals that are factors of $(2)$ or $(3)$.



          Dedekind's criterion. Dedekind's criterion is a way of factorising $(3)$ as a product of primes.



          As you pointed out, we have the factorisation
          $$ x^2 - x + frac 842 equiv x(x-1) mod 3,$$



          so Dedekind's criterion says that
          $$ (3) = (3, theta)(3, theta - 1)$$
          is the prime factorisation of $(3)$ in $mathbb Z[theta]$.



          Whether $mathbb Z[theta]$ is a UFD. This is equivalent to asking whether the ideal class group is trivial, i.e. whether all ideals are principal.



          Why don't we check whether $(3, theta)$ is principal? The answer must be "no". Note that $(3, theta)$ has norm $3$. If it was principal, then there would exist $x, y in mathbb Z$ such that $(3, theta) = (x + ytheta)$. But then
          $$ 3 = N(3, theta) = N(x + ytheta) = (x + tfrac 1 2 y)^2 + 83(tfrac 1 2 y)^2,$$
          which is impossible.



          So $mathbb Z[theta]$ is not a principal ideal domain, and hence, is not a unique factorisation domain.






          share|cite|improve this answer









          $endgroup$












          • $begingroup$
            Thanks that helpful, that what I was looking for
            $endgroup$
            – Ameryr
            4 hours ago














          4












          4








          4





          $begingroup$

          You seem to have touched upon several different ideas here.



          Generators of the ideal $(3)$. Usually, when you talk about the generators of $(3)$, you mean its generators as an abelian group.



          Define $theta := tfrac 1 2 (1 + sqrt-83)$. We know that the ring of integers $mathbb Z[theta]$ is generated by $1, theta $ as an abelian group, so $(3)$ is generated by $3, 3theta $ as an abelian group.



          But having read the remainder of your question, it looks like this is not what you're after! What you're really after are the generators of the ideal class group for $mathbb Z[theta]$...



          Minkowski constant. The fact that the Minkowski constant is $5.8$ implies that the ideal class group is generated by prime ideals that are factors of $(2)$ or $(3)$.



          Dedekind's criterion. Dedekind's criterion is a way of factorising $(3)$ as a product of primes.



          As you pointed out, we have the factorisation
          $$ x^2 - x + frac 842 equiv x(x-1) mod 3,$$



          so Dedekind's criterion says that
          $$ (3) = (3, theta)(3, theta - 1)$$
          is the prime factorisation of $(3)$ in $mathbb Z[theta]$.



          Whether $mathbb Z[theta]$ is a UFD. This is equivalent to asking whether the ideal class group is trivial, i.e. whether all ideals are principal.



          Why don't we check whether $(3, theta)$ is principal? The answer must be "no". Note that $(3, theta)$ has norm $3$. If it was principal, then there would exist $x, y in mathbb Z$ such that $(3, theta) = (x + ytheta)$. But then
          $$ 3 = N(3, theta) = N(x + ytheta) = (x + tfrac 1 2 y)^2 + 83(tfrac 1 2 y)^2,$$
          which is impossible.



          So $mathbb Z[theta]$ is not a principal ideal domain, and hence, is not a unique factorisation domain.






          share|cite|improve this answer









          $endgroup$



          You seem to have touched upon several different ideas here.



          Generators of the ideal $(3)$. Usually, when you talk about the generators of $(3)$, you mean its generators as an abelian group.



          Define $theta := tfrac 1 2 (1 + sqrt-83)$. We know that the ring of integers $mathbb Z[theta]$ is generated by $1, theta $ as an abelian group, so $(3)$ is generated by $3, 3theta $ as an abelian group.



          But having read the remainder of your question, it looks like this is not what you're after! What you're really after are the generators of the ideal class group for $mathbb Z[theta]$...



          Minkowski constant. The fact that the Minkowski constant is $5.8$ implies that the ideal class group is generated by prime ideals that are factors of $(2)$ or $(3)$.



          Dedekind's criterion. Dedekind's criterion is a way of factorising $(3)$ as a product of primes.



          As you pointed out, we have the factorisation
          $$ x^2 - x + frac 842 equiv x(x-1) mod 3,$$



          so Dedekind's criterion says that
          $$ (3) = (3, theta)(3, theta - 1)$$
          is the prime factorisation of $(3)$ in $mathbb Z[theta]$.



          Whether $mathbb Z[theta]$ is a UFD. This is equivalent to asking whether the ideal class group is trivial, i.e. whether all ideals are principal.



          Why don't we check whether $(3, theta)$ is principal? The answer must be "no". Note that $(3, theta)$ has norm $3$. If it was principal, then there would exist $x, y in mathbb Z$ such that $(3, theta) = (x + ytheta)$. But then
          $$ 3 = N(3, theta) = N(x + ytheta) = (x + tfrac 1 2 y)^2 + 83(tfrac 1 2 y)^2,$$
          which is impossible.



          So $mathbb Z[theta]$ is not a principal ideal domain, and hence, is not a unique factorisation domain.







          share|cite|improve this answer












          share|cite|improve this answer



          share|cite|improve this answer










          answered 5 hours ago









          Kenny WongKenny Wong

          20k21442




          20k21442











          • $begingroup$
            Thanks that helpful, that what I was looking for
            $endgroup$
            – Ameryr
            4 hours ago

















          • $begingroup$
            Thanks that helpful, that what I was looking for
            $endgroup$
            – Ameryr
            4 hours ago
















          $begingroup$
          Thanks that helpful, that what I was looking for
          $endgroup$
          – Ameryr
          4 hours ago





          $begingroup$
          Thanks that helpful, that what I was looking for
          $endgroup$
          – Ameryr
          4 hours ago












          3












          $begingroup$

          The minimal polynomial of $alpha=frac1+sqrt-832$ is $f = x^2 - x + 84/4 = x^2 - x + 21$. Modulo $3$ this factors as $$f equiv x^2 - x = x(x-1) pmod 3,$$
          so by the Kummer-Dedekind theorem the ideal $(3)$ factors into primes in $mathbbZbigg[frac1+sqrt-832bigg]$ as $(3) = (3, alpha)(3, alpha-1)$.



          The ideal $(3)$ is principal, generated by $3$. You can show that the prime factors are not principal, e.g. using the field norm:



          If $(3,alpha) = (beta)$ then $N(beta)$ divides both $N(3) = 3^2$ and $N(alpha) = f(0) = 21 = 3cdot 7$, so $N(beta) = 3$.



          If we write $beta = a+balpha$ then $$N(beta)=(a+balpha)(a+b(1-alpha)) = ldots = a^2+ab + 21b^2.$$



          The equation $a^2+ab + 21b^2 = 3$ is an ellipse in the $(a,b)$-plane without integral points, so there is no such $beta$.






          share|cite|improve this answer









          $endgroup$








          • 1




            $begingroup$
            How could you determine that the ellipse has no integral points?
            $endgroup$
            – Ameryr
            4 hours ago






          • 1




            $begingroup$
            I plotted it (with a grid in the background). It's a very small ellipse, so there are very few candidates for integral points, and you can see that it misses them all.
            $endgroup$
            – Ricardo Buring
            4 hours ago















          3












          $begingroup$

          The minimal polynomial of $alpha=frac1+sqrt-832$ is $f = x^2 - x + 84/4 = x^2 - x + 21$. Modulo $3$ this factors as $$f equiv x^2 - x = x(x-1) pmod 3,$$
          so by the Kummer-Dedekind theorem the ideal $(3)$ factors into primes in $mathbbZbigg[frac1+sqrt-832bigg]$ as $(3) = (3, alpha)(3, alpha-1)$.



          The ideal $(3)$ is principal, generated by $3$. You can show that the prime factors are not principal, e.g. using the field norm:



          If $(3,alpha) = (beta)$ then $N(beta)$ divides both $N(3) = 3^2$ and $N(alpha) = f(0) = 21 = 3cdot 7$, so $N(beta) = 3$.



          If we write $beta = a+balpha$ then $$N(beta)=(a+balpha)(a+b(1-alpha)) = ldots = a^2+ab + 21b^2.$$



          The equation $a^2+ab + 21b^2 = 3$ is an ellipse in the $(a,b)$-plane without integral points, so there is no such $beta$.






          share|cite|improve this answer









          $endgroup$








          • 1




            $begingroup$
            How could you determine that the ellipse has no integral points?
            $endgroup$
            – Ameryr
            4 hours ago






          • 1




            $begingroup$
            I plotted it (with a grid in the background). It's a very small ellipse, so there are very few candidates for integral points, and you can see that it misses them all.
            $endgroup$
            – Ricardo Buring
            4 hours ago













          3












          3








          3





          $begingroup$

          The minimal polynomial of $alpha=frac1+sqrt-832$ is $f = x^2 - x + 84/4 = x^2 - x + 21$. Modulo $3$ this factors as $$f equiv x^2 - x = x(x-1) pmod 3,$$
          so by the Kummer-Dedekind theorem the ideal $(3)$ factors into primes in $mathbbZbigg[frac1+sqrt-832bigg]$ as $(3) = (3, alpha)(3, alpha-1)$.



          The ideal $(3)$ is principal, generated by $3$. You can show that the prime factors are not principal, e.g. using the field norm:



          If $(3,alpha) = (beta)$ then $N(beta)$ divides both $N(3) = 3^2$ and $N(alpha) = f(0) = 21 = 3cdot 7$, so $N(beta) = 3$.



          If we write $beta = a+balpha$ then $$N(beta)=(a+balpha)(a+b(1-alpha)) = ldots = a^2+ab + 21b^2.$$



          The equation $a^2+ab + 21b^2 = 3$ is an ellipse in the $(a,b)$-plane without integral points, so there is no such $beta$.






          share|cite|improve this answer









          $endgroup$



          The minimal polynomial of $alpha=frac1+sqrt-832$ is $f = x^2 - x + 84/4 = x^2 - x + 21$. Modulo $3$ this factors as $$f equiv x^2 - x = x(x-1) pmod 3,$$
          so by the Kummer-Dedekind theorem the ideal $(3)$ factors into primes in $mathbbZbigg[frac1+sqrt-832bigg]$ as $(3) = (3, alpha)(3, alpha-1)$.



          The ideal $(3)$ is principal, generated by $3$. You can show that the prime factors are not principal, e.g. using the field norm:



          If $(3,alpha) = (beta)$ then $N(beta)$ divides both $N(3) = 3^2$ and $N(alpha) = f(0) = 21 = 3cdot 7$, so $N(beta) = 3$.



          If we write $beta = a+balpha$ then $$N(beta)=(a+balpha)(a+b(1-alpha)) = ldots = a^2+ab + 21b^2.$$



          The equation $a^2+ab + 21b^2 = 3$ is an ellipse in the $(a,b)$-plane without integral points, so there is no such $beta$.







          share|cite|improve this answer












          share|cite|improve this answer



          share|cite|improve this answer










          answered 5 hours ago









          Ricardo BuringRicardo Buring

          1,98121437




          1,98121437







          • 1




            $begingroup$
            How could you determine that the ellipse has no integral points?
            $endgroup$
            – Ameryr
            4 hours ago






          • 1




            $begingroup$
            I plotted it (with a grid in the background). It's a very small ellipse, so there are very few candidates for integral points, and you can see that it misses them all.
            $endgroup$
            – Ricardo Buring
            4 hours ago












          • 1




            $begingroup$
            How could you determine that the ellipse has no integral points?
            $endgroup$
            – Ameryr
            4 hours ago






          • 1




            $begingroup$
            I plotted it (with a grid in the background). It's a very small ellipse, so there are very few candidates for integral points, and you can see that it misses them all.
            $endgroup$
            – Ricardo Buring
            4 hours ago







          1




          1




          $begingroup$
          How could you determine that the ellipse has no integral points?
          $endgroup$
          – Ameryr
          4 hours ago




          $begingroup$
          How could you determine that the ellipse has no integral points?
          $endgroup$
          – Ameryr
          4 hours ago




          1




          1




          $begingroup$
          I plotted it (with a grid in the background). It's a very small ellipse, so there are very few candidates for integral points, and you can see that it misses them all.
          $endgroup$
          – Ricardo Buring
          4 hours ago




          $begingroup$
          I plotted it (with a grid in the background). It's a very small ellipse, so there are very few candidates for integral points, and you can see that it misses them all.
          $endgroup$
          – Ricardo Buring
          4 hours ago

















          draft saved

          draft discarded
















































          Thanks for contributing an answer to Mathematics Stack Exchange!


          • Please be sure to answer the question. Provide details and share your research!

          But avoid


          • Asking for help, clarification, or responding to other answers.

          • Making statements based on opinion; back them up with references or personal experience.

          Use MathJax to format equations. MathJax reference.


          To learn more, see our tips on writing great answers.




          draft saved


          draft discarded














          StackExchange.ready(
          function ()
          StackExchange.openid.initPostLogin('.new-post-login', 'https%3a%2f%2fmath.stackexchange.com%2fquestions%2f3197383%2fdetermine-the-generator-of-an-ideal-of-ring-of-integers%23new-answer', 'question_page');

          );

          Post as a guest















          Required, but never shown





















































          Required, but never shown














          Required, but never shown












          Required, but never shown







          Required, but never shown

































          Required, but never shown














          Required, but never shown












          Required, but never shown







          Required, but never shown







          Popular posts from this blog

          Log på Navigationsmenu

          Wonderful Copenhagen (sang) Eksterne henvisninger | NavigationsmenurSide på frankloesser.comWonderful Copenhagen

          Detroit Tigers Spis treści Historia | Skład zespołu | Sukcesy | Członkowie Baseball Hall of Fame | Zastrzeżone numery | Przypisy | Menu nawigacyjneEncyclopedia of Detroit - Detroit TigersTigers Stadium, Detroit, MITigers Timeline 1900sDetroit Tigers Team History & EncyclopediaTigers Timeline 1910s1935 World Series1945 World Series1945 World Series1984 World SeriesComerica Park, Detroit, MI2006 World Series2012 World SeriesDetroit Tigers 40-Man RosterDetroit Tigers Coaching StaffTigers Hall of FamersTigers Retired Numberse